2006 SMT/General Problems/Problem 13

Revision as of 17:20, 13 January 2020 by Dividend (talk | contribs) (Created page with "==Solution== This solution does require quite a bit of guessing and testing, but if we notice that we want the numbers to be relatively close so that no term "dominates" the...")
(diff) ← Older revision | Latest revision (diff) | Newer revision → (diff)

Solution

This solution does require quite a bit of guessing and testing, but if we notice that we want the numbers to be relatively close so that no term "dominates" the sum, we see that the solution is $\boxed{123-45-67+89=100}$